Lässt sich Verschränkung als Folge von Erhaltungssätzen erklären?

Dieser Artikel im Magazin NewScientist (Abonnement erforderlich) beschreibt die Verschränkung von Photonen, indem sie durch einen halbversilberten Spiegel geleitet werden.

http://www.newscientist.com/article/mg21929282.100-quantum-weirdness-the-battle-for-the-basis-of-reality.html

Es spricht über die "Seltsamkeit" der Verschränkung und wie dies bedeutet, dass sich entweder Informationen schneller als mit Lichtgeschwindigkeit fortbewegen oder andere grundlegende Annahmen über die Realität falsch sind, weil die Spins der austretenden Photonen immer korreliert sind, wenn sie gemessen werden.

Meine Frage ist nur, was ist daran so seltsam? Ich verstehe es nicht ... Es scheint mir nur eine weitere Variante der Erhaltungsgesetze zu sein. Es wäre so, als würde man sagen, dass es für ein Photon in Paarproduktion "komisch" ist, immer Teilchen mit entgegengesetzter Ladung zu erzeugen. Das ist eine Selbstverständlichkeit ... Gibt es etwas an dem Timing, das alle Federn zerzaust?

Ich glaube, ich vermisse etwas.

Ich denke, was Sie vermissen, ist, dass es keine lokalen versteckten Variablen gibt. Das bedeutet, dass der Spin eines verschränkten Teilchens nicht nur unbekannt ist, sondern erst festgelegt wird, wenn jemand den Spin seines Paares misst. Das ist haarsträubend, weil es bedeutet, dass es eine Kommunikation zwischen verschränkten Teilchen mit Überlichtgeschwindigkeit geben würde, die besagt, dass eines gemessen wurde und den Spin des anderen bestimmt.
Woher wissen wir, dass es nicht gesetzt ist, wenn wir nicht sagen können, welchen Wert es hat, bis wir es messen?
IIRC, normalerweise interpretieren wir eine Observable so, dass sie keinen sinnvollen Wert hat, bis wir sie messen. Verzögerte Quantenlöscher und dergleichen sind anders schwer zu erklären.
@Jim man könnte genau dasselbe für die Erzeugung von Paarpartikeln sagen. Bis zum Beispiel ein Magnetfeld angelegt wird, kann man ein Positron nicht von einem Elektron unterscheiden. So? Ich bin da bei John. In den Regeln des QM gibt es nichts Mysteriöses. Zu viel Philosophie. Könnten Sie die Notwendigkeit versteckter Variablen deutlicher machen, um diese einfachen Situationen zu erklären?
@annav Ehrlich gesagt fühle ich mich von Ihrer Denkweise angezogen, aber die Forschung hat mir gezeigt, dass die Hauptprobleme aus einem Mangel an lokalen versteckten Variablen resultieren. In den meisten Fällen ist es dasselbe, aber im Gegensatz zur Ladung ist der Spin eine lokale Variable. Wenn Sie also in Situationen wie dem Experiment verschränkte Teilchen verwenden, um die Bellsche Ungleichung zu beweisen, erhalten Sie am Ende ein nicht klassisches Ergebnis.
Seltsam wird das erst, wenn die Quelle Überlagerungen erzeugt. Wenn + für das Drehen in eine Richtung und - für das Drehen in die andere Richtung steht, dann | + ist ein vollkommen gültiger Zustand, der den Erhaltungsgesetzen gehorcht und nicht verstrickt ist. So ist | + . Ir
verwandt: physical.stackexchange.com/questions/68147/… . Wie dort diskutiert, ist Verschränkung erforderlich, wenn Erhaltungssätze exakt und nicht nur statistisch gelten sollen. Dies ist eine gültige Erklärung dafür, warum Verschränkung existieren muss. Es spricht jedoch nicht die grundlegende Seltsamkeit der Verschränkung an.

Antworten (3)

Vielleicht ist das magische Quadratspiel Mermin-Peres eine gute Demonstration dafür, warum Verschränkung so rätselhaft ist. Es gibt drei Spieler, von denen zwei (z. B. A und B) verschränkte Zustände haben.

A und B dürfen ihre Strategie im Voraus kommunizieren und vereinbaren, aber sie können nicht kommunizieren, sobald das Spiel im Gange ist.

Da ist ein 3 × 3 Netz. Sie können A nach einer beliebigen Spalte des Rasters fragen (aber nur eine), und Sie können B nach einer beliebigen Zeile des Rasters fragen (aber nur eine). Die Regeln sind, dass A und B ihren Zellen im Gitter 0 und 1 zuweisen müssen, sie müssen sich auf die Zelle einigen, in der sich Zeile und Spalte schneiden, und die Anzahl der 1 in einer Spalte ist immer ungerade, aber die Anzahl der 1 in eine Reihe ist immer gerade.

Wenn Sie beispielsweise nach Spalte 1 und Zeile 2 fragen, könnten sie Folgendes zurückgeben:

A      B

1xx    xxx  
0xx    011  
0xx    xxx

Wenn sie verschränkte Zustände haben, können A und B immer gewinnen.

"Einfach", sagen Sie, "A und B haben sich im Voraus darauf geeinigt, welche Zellen 0 und 1 haben, und sie geben einfach diese Werte zurück."

Aber wenn das ihre Strategie ist, hat das Hauptgitter eine gerade oder eine ungerade Anzahl von Einsen?

"Was ist daran so seltsam?" Dies war im Wesentlichen der Punkt des EPR-Arguments von 1935, das besagt, dass es dumm ist, alle Arten von Quantenverrücktheit zu postulieren, wenn Korrelationen einfach durch die Annahme erklärt werden könnten, dass Teilchen bestimmte Eigenschaften haben. Wenn ich je ein Paar Socken in zwei Schachteln stecke und die Schachteln weit auseinander schiebe, dann ist es keine Überraschung, dass die Socken übereinstimmen, wenn die Schachteln geöffnet werden – es handelt sich nicht um Verrücktheit oder Überlichtgeschwindigkeiten.

Das EPR-Argument wurde weitgehend ignoriert, nicht so sehr, wie es scheint, weil die Leute ein gutes Gegenargument hatten, sondern weil die Leute Quantenverrücktheit wirklich, wirklich mögen. In den frühen 1960er Jahren griff John Bell das EPR-Argument wieder auf. Er stellte fest, dass das EPR-Argument zwar für einfache Fälle wie die Socken funktionierte, bei etwas komplizierteren Setups jedoch auf Probleme stieß.

Zwei verschränkte Teilchen werden von einer zentralen Quelle emittiert, um von Alice und Bob gemessen zu werden, die in einiger Entfernung voneinander entfernt sind und jeweils ein Messgerät mit einem Knopf mit drei Einstellungen A, B und C haben und eine Null oder eine Eins liefern als Ergebnis der Messung.

Beim Vergleich der Ergebnisse für lange Verbindungsmessungen stellen Alice und Bob fest, dass die Ergebnisse bei gleichen Einstellungen immer gleich sind. Wenn sich die Einstellungen um eine Position unterscheiden (also eine hat A und die andere B, oder eine hat B und die andere C), dann unterscheiden sich die Ergebnisse um 1/7 der Zeit. Etwas Nachdenken führt dann zu dem Ergebnis, wenn man Einstellung A hat und die andere Einstellung C, dann können sich die Ergebnisse höchstens in 2/7 der Zeit unterscheiden.

Das Problem ist, dass dies in der Realität nicht der Fall ist. Ein Quantenexperiment kann so aufgebaut werden, dass sich die Ergebnisse in der Hälfte der Zeit unterscheiden. Es ist dieses Ergebnis, das als Erklärung Quantenverrücktheit erfordert

Für mein Geld ist das am einfachsten zu verstehende Beispiel dafür, warum Verschränkung seltsam ist, Mermins Version von Bells Theorem. Es ist auf eine schöne, nicht-technische Weise geschrieben

N. David Mermin, „Quantenmysterien für Jedermann“ . Zeitschrift für Philosophie , Bd. 78, Nr. 7. (1981), 397-408.

Hier die Grundidee: Wir haben einen Versuchsaufbau, der aus drei Teilen besteht. Zwei der Teile sind "Detektoren"; sie bestehen jeweils aus einem Schalter, der in eine von drei Positionen (A, B oder C) gebracht werden kann, und einer Anzeige, die entweder „JA“ oder „NEIN“ leuchten kann. Der dritte Teil ist ein "Sender", der zwischen den beiden Detektoren stationiert ist. Immer wenn wir einen Knopf auf dem Sender drücken, sendet er zwei Teilchen aus, eines an jeden der Detektoren. Jeder Melder leuchtet dann mit „JA“ oder „NEIN“ auf.

Wenn wir dieses Experiment viele Male durchführen und die Einstellungen der Schalter an beiden Detektoren variieren, finden wir Folgendes:

  1. Jeder einzelne Detektor blinkt 50 % der Zeit „JA“ und 50 % der Zeit „NEIN“. Dies ist unabhängig von der Schalterstellung des Detektors (A, B oder C).

  2. Wenn die Schalter an den beiden Detektoren beide auf die gleiche Position (A, B, C) eingestellt sind, stimmen die Ergebnisse der beiden Detektoren immer überein: Sie blinken beide „JA“ oder beide blinken „NEIN“.

  3. Wenn die Schalter an den Detektoren auf unterschiedliche Einstellungen eingestellt sind, stimmen die Ergebnisse der beiden Detektoren überein 1 4 der Zeit (dh beide "JA" oder beide "NEIN"). Sie stimmen nicht überein 3 4 der ganzen Zeit.

Wie sind diese Ergebnisse zu erklären? Ergebnis Nr. 2 könnte ziemlich einfach erklärt werden. Der Sender könnte einfach zwei Teilchen mit einer Reihe von "Anweisungen" aussenden, die ihm sagen, was zu tun ist, wenn er jeden Detektor erreicht. Zum Beispiel könnten die Partikel die Anweisungen tragen: „Lass den Detektor ‚NEIN‘ blinken, wenn der Schalter auf Position A oder B steht, und ‚JA‘ blinken, wenn er auf Position C steht.“ Mit anderen Worten, die Partikel haben bestimmte Eigenschaften, wenn sie emittiert werden, und die Detektoren messen einfach diese Eigenschaften.

Aber denken wir an Ergebnis Nr. 3. Wenn wir davon ausgehen, dass die Partikel beide eindeutige „Anweisungen“ haben, dann gibt es acht mögliche Anweisungssätze, die ihnen gegeben werden können:

 A B C
-------
 Y Y Y
 N N N
 Y Y N
 Y N Y
 N Y Y
 Y N N
 N Y N
 N N Y

Zum Beispiel entsprechen die oben erwähnten Anweisungen (lassen Sie den Detektor „NEIN“ blinken, wenn sich der Schalter in Position A oder B befindet, und „JA“ blinken, wenn er sich in Position C befindet) der letzten Zeile der Tabelle.

Bei den ersten beiden Befehlssätzen stimmen die Detektoren immer überein, wenn ihre Schalter auf unterschiedliche Einstellungen eingestellt sind. Bei den anderen sechs Befehlssätzen stimmen die Detektoren in 1/3 der Fälle zu und in 2/3 der Fälle nicht. Die vom Sender gesendeten Partikel können aufgrund von Ergebnis Nr. 1 nicht jedes Mal die gleichen Anweisungen haben; Vielmehr muss es jedes Mal einen anderen Satz von Anweisungen auswählen. Unabhängig davon, wie diese "Anweisungen" ausgewählt werden, würden wir jedoch erwarten, dass die Detektoren in mindestens 1/3 der Fälle zustimmen würden; jeder Anweisungssatz führt entweder zu 100 % Zustimmung oder 33 % Zustimmung.

Aber Ergebnis Nr. 3 sagt, dass die Detektoren zustimmen 1 4 der Zeit und 1 4 < 1 3 . Wo ist also unsere Argumentation schief gelaufen?


SIDEBAR: Die Geräte können (im Prinzip) wie folgt aufgebaut sein: Der Sender erzeugt zwei Elektronen in einem verschränkten Zustand | | | | , und sendet ein Elektron an jeden Detektor. Die Detektoren bestehen aus einem Stern-Gerlach-Apparat, der um die Laufrichtung der Elektronen gedreht werden kann; die Einstellungen A, B und C setzen diesen Winkel entweder auf 0°, 120° oder 240°. Einer der Detektoren blinkt "JA", wenn ein Elektron zum Nordpol seines Magneten abgelenkt wird, und "NEIN", wenn ein Elektron zum Südpol abgelenkt wird. Der andere Detektor verwendet die entgegengesetzte Konvention. Die Standard-Quantenmechanik sagt voraus, dass die beiden Geräte nur einen Bruchteil übereinstimmen cos 2 θ der Zeit, wo θ ist der Winkel zwischen ihren Magneten. In diesem Fall, cos 2 θ = 1 4 wenn die Detektoren auf unterschiedliche Winkel eingestellt sind, und cos 2 θ = 1 wenn sie auf den gleichen Winkel eingestellt sind.